微觀經濟學

擬線性效用:帕累托最優意味著總效用最大化?

  • January 24, 2017

我讀到,如果我們對所有消費者都有擬線性效用,那麼任何帕累托最優分配都會使所有消費者的效用水平之和最大化。那是:

$ \textbf{What we know:} $

$$ 1)\quad u^i(m^i,x^i)=m^i+\phi^i(x^i); \quad \forall i=1,…,I $$ $$ 2)\quad\phi^i(;);\text{is continous and strictly increasing (but not necessarily differentiable)} $$ $$ 3)\quad \text{An allocation,},x, \text{satisfies};\neg,\exists,\hat{x}; s.t. ;\hat{m}^i+\phi^i(\hat{x}^i)\geq m^i+\phi(x^i);\forall i $$ $$ \text{and} \quad \hat{m}^i+\phi^i(\hat{x}^i)> m^i+\phi(x^i),\text{for some},i $$ $ \textbf{What to show:} $

$$ x;\text{solves};max\sum_{i=1}^Im^i+\phi^i(x^i) $$ 任何人都可以提供證據嗎?任何幫助將不勝感激!

$ \textbf{Edit:}, $ 我不知道這是否是正確的道路,但通過嚴格遞增的屬性 $ \phi(,) $ ,偏好滿足局部不滿足,這意味著它們滿足第一福利定理。現在,如果我能弄清楚所有帕累托最優分配是否都是具有擬線性效用的競爭均衡,我可能會有所收穫!

**編輯:**邊緣情況很糟糕;看評論。另見 MWG 第 10 章 C、D 部分。


認為 $ (\vec x^, \vec m^) $ 解決

$$ \max \sum^I_{i=1} m_i + \phi_i(x_i) $$ 但不是帕累托最優。

$$ \begin{align} \implies \exists \ (x_i’, m_i’) \quad \text{s.t.} \quad & u_i(x_i’, m_i’) \geq u_i(x_i^, m_i^) \quad \forall \ i = 1,\cdots,I \ & u_i(x_i’, m_i’) > u_i(x_i^, m_i^) \quad \text{for some} \ i \end{align} $$ $$ \implies \sum^I_{i=1} m’_i + \phi_i(x’i) > \sum^I{i=1} m^_i + \phi_i(x^_i) $$ 這是一個矛盾。如果我們有一個效用最大化問題的解決方案,它一定是帕累托最優的。

(請注意,這來自於 $ \phi(\cdot) $ )


認為 $ (\vec x^, \vec m^) $ 是可行的帕累托最優分配,但不解決

$$ \max \sum^I_{i=1} m_i + \phi_i(x_i) $$ 因為我們對待 $ m_i $ 作為計價單位和 $ \phi_i(\cdot) $ 嚴格增加,我們知道 $ u_i(\cdot) $ 是局部不滿足的。帕累托分配應該是可行的。

$$ \exists \ (x_i’, m_i’) \quad \text{s.t.} \quad \sum^I_{i=1} m’_i + \phi_i(x’i) > \sum^I{i=1} m^_i + \phi_i(x^i)\ \implies \boxed{ \sum^I{i=1} \phi_i(x’i) > \sum^I{i=1} \phi_i(x^*_i)} $$ 如果這是真的,因為這種替代分配只是給了個人更多 $ x $ ,在其他條件相同的情況下,替代分配是不可行的。所以我們會有矛盾。

如果這是真的,因為在替代分配中,其他人被分配了更多 $ x $ 並且只有一個人被分配得更少,那麼原始分配就不是帕累托最優的。假設是。如果您採用原始分配並轉移 $ x $ 在新分配的方式中,您將需要相應的計價商品交易, $ m $ , 留住誰輸了 $ x $ 至少在相同的效用水平。但是僅以計價商品進行交易永遠不會改變總效用的總和。從原來的分配,如果你可以交易 $ m $ 為了 $ x $ 在不傷害任何人的情況下讓某人過得更好,你沒有處於帕累托最優,如果你不能交易 $ m $ 為了 $ x $ 為了讓某人過得更好,你不能增加總效用,這意味著原始分配是最大化問題的解決方案。

無論您如何重新排列,此邏輯都適用 $ x $ 多人之間。

$ \square $

引用自:https://economics.stackexchange.com/questions/11033